LSAT and Law School Admissions Forum

Get expert LSAT preparation and law school admissions advice from PowerScore Test Preparation.

 cgleeson
  • Posts: 75
  • Joined: Feb 13, 2022
|
#93996
Hi Beth,

In reversing the statement how do I go about that?
"it is not necessary to officially confirm scientific studies before they are published because bad science will be disconfirmed later."

Am I stating "bad science will be disconfirmed later because no scientific studies are confirmed before they are published" ?

Either way I kind of see your point, it does not make any sense. Thank you so much

Chris 8-)

Beth Hayden wrote: Tue Feb 22, 2022 7:28 pm Hi C,

This is definitely a tricky question to catch the conclusion! Remember, an argument can have multiple conclusions, but only one is the main conclusion (the other are intermediate conclusions). What's the difference? Well, the main conclusion is supported by every other statement in the stimulus but is not used to support something else.
An intermediate conclusion is in turn used to support the main conclusion. If you narrow it down to two statements and you're not sure which is the main conclusion, just ask: is one statement used to help prove the other statement? One trick I use is to insert the word "because" after one of the statements. If X is true because of Y, Y is not the main conclusion.

Here, the argument says that it is not necessary to officially confirm scientific studies before they are published because bad science will be disconfirmed later. If you reverse those statements, it makes no sense. The last sentence is used to support the first sentence.

I hope that helps!
Beth
User avatar
 Beth Hayden
PowerScore Staff
  • PowerScore Staff
  • Posts: 123
  • Joined: Sep 04, 2021
|
#94259
Hi C,

If you switch the statements you would get:

Bad science will be disconfirmed later because it is not necessary to officially confirm scientific studies before they are published. That doesn't make any sense, right? The claim that bad science will be uncovered in the future is a reason why the author thinks you shouldn't have to officially confirm studies before publishing them.

Let me know if you have any further questions on that!

Best,
Beth
 cgleeson
  • Posts: 75
  • Joined: Feb 13, 2022
|
#94268
Hi Beth,
In regards to the "because" test to confirm the conclusion if X is not true because of Y
, Y is not the conclusion. Then it could be true that X is the conclusion and Y may be a subsidiary conclusion.
If I may ask one other question, on the "Negation Test" can I use that with all necessary assumption questions? I would not however use that test on a sufficient assumption or strengthen question, is that correct?
Many thanks Beth,
Chris 8-)

Beth Hayden wrote: Tue Feb 22, 2022 7:28 pm Hi C,

This is definitely a tricky question to catch the conclusion! Remember, an argument can have multiple conclusions, but only one is the main conclusion (the other are intermediate conclusions). What's the difference? Well, the main conclusion is supported by every other statement in the stimulus but is not used to support something else.
An intermediate conclusion is in turn used to support the main conclusion. If you narrow it down to two statements and you're not sure which is the main conclusion, just ask: is one statement used to help prove the other statement? One trick I use is to insert the word "because" after one of the statements. If X is true because of Y, Y is not the main conclusion.

Here, the argument says that it is not necessary to officially confirm scientific studies before they are published because bad science will be disconfirmed later. If you reverse those statements, it makes no sense. The last sentence is used to support the first sentence.

I hope that helps!
Beth
 Adam Tyson
PowerScore Staff
  • PowerScore Staff
  • Posts: 5387
  • Joined: Apr 14, 2011
|
#94284
That's correct, Chris!
User avatar
 danielnoah820
  • Posts: 1
  • Joined: Jan 22, 2023
|
#98994
Thank you for your prompt responses,
But I thought that on weaken questions, premises go unchallenged and just the reasoning that comes from premises to establish the conclusion is suspect. In this stimulus the premise is challenged
User avatar
 Jeff Wren
PowerScore Staff
  • PowerScore Staff
  • Posts: 657
  • Joined: Oct 19, 2022
|
#99004
Hi Daniel,

You absolutely can weaken an argument by attacking one or more of the premises of the argument.

There are two main ways of weakening an argument.

1. To directly attack a premise (since the argument is relying on those premises, an answer that attacks a premise definitely attacks the support for the argument).

2. To show that the conclusion does not follow from the premises that are given (i.e. to show a logical gap in the argument).

While the second way is more common on the LSAT, both do come up.
User avatar
 Dancingbambarina
  • Posts: 49
  • Joined: Mar 30, 2024
|
#110349
Hi,

Why is it that C does not help indictethere is no reason: afterall, if scientists are under pressure to have work checked, there is no reason but strongarming that they do so
 Robert Carroll
PowerScore Staff
  • PowerScore Staff
  • Posts: 1819
  • Joined: Dec 06, 2013
|
#110487
Dancingbambarina,

But, as noted in the first post, answer choice (C) strengthens the argument. If scientists are under pressure to make their work accessible to replication, then they are acting in such a way that the scrutiny of replication will be more readily applied to their work. This enhances the author's case that official confirmation is not as necessary, because the scrutiny of replication will be more easily applied, reducing the possible ill effects of hasty publication.

Robert Carroll

Get the most out of your LSAT Prep Plus subscription.

Analyze and track your performance with our Testing and Analytics Package.